What is Angular: Definition and 999 Discussions

In physics, angular momentum (rarely, moment of momentum or rotational momentum) is the rotational equivalent of linear momentum. It is an important quantity in physics because it is a conserved quantity—the total angular momentum of a closed system remains constant.
In three dimensions, the angular momentum for a point particle is a pseudovector r × p, the cross product of the particle's position vector r (relative to some origin) and its momentum vector; the latter is p = mv in Newtonian mechanics. Unlike momentum, angular momentum depends on where the origin is chosen, since the particle's position is measured from it.
Just as for angular velocity, there are two special types of angular momentum of an object: the spin angular momentum is the angular momentum about the object's centre of mass, while the orbital angular momentum is the angular momentum about a chosen center of rotation. The total angular momentum is the sum of the spin and orbital angular momenta. The orbital angular momentum vector of a point particle is always parallel and directly proportional to its orbital angular velocity vector ω, where the constant of proportionality depends on both the mass of the particle and its distance from origin. The spin angular momentum vector of a rigid body is proportional but not always parallel to the spin angular velocity vector Ω, making the constant of proportionality a second-rank tensor rather than a scalar.
Angular momentum is an extensive quantity; i.e. the total angular momentum of any composite system is the sum of the angular momenta of its constituent parts. For a continuous rigid body or a fluid the total angular momentum is the volume integral of angular momentum density (i.e. angular momentum per unit volume in the limit as volume shrinks to zero) over the entire body.
Torque can be defined as the rate of change of angular momentum, analogous to force. The net external torque on any system is always equal to the total torque on the system; in other words, the sum of all internal torques of any system is always 0 (this is the rotational analogue of Newton's Third Law). Therefore, for a closed system (where there is no net external torque), the total torque on the system must be 0, which means that the total angular momentum of the system is constant. The conservation of angular momentum helps explain many observed phenomena, for example the increase in rotational speed of a spinning figure skater as the skater's arms are contracted, the high rotational rates of neutron stars, the Coriolis effect, and the precession of gyroscopes. In general, conservation limits the possible motion of a system but does not uniquely determine it.
In quantum mechanics, angular momentum (like other quantities) is expressed as an operator, and its one-dimensional projections have quantized eigenvalues. Angular momentum is subject to the Heisenberg uncertainty principle, implying that at any time, only one projection (also called "component") can be measured with definite precision; the other two then remain uncertain. Because of this, the axis of rotation of a quantum particle is undefined. Quantum particles do possess a type of non-orbital angular momentum called "spin", but this angular momentum does not correspond to a spinning motion.

View More On Wikipedia.org
  1. Isolde Wilde

    Are angular & vertical velocity the same if the objects are connected

    think of a engine. it has a flywheel and a rod connected to it. a string had been totally wrapped around the rod and a mass is hanged from the very end of the rod. the system is in equilibrium. but as the engine starts to rotate, the rod with rotate as well and cause the hanged object to go...
  2. Boltzman Oscillation

    Law of conservation of angular momentum

    Given the figure, how can i arrive to this formula knowing that angular momentum is conserved? I know that p = mv and L = p x r. So the initial momentum will be L1 = mV x R and the final momentum will be L2 = mv x r. I am not sure how R will equal to b since the distance between the...
  3. Q

    I CSCO of Total Angular momentum

    I understand that in a system composed of two articles, the total angular momentum is: J = J1 + J2 From the operators: J^2, Jz, J1z, J2z,J^21z,J^22z, I get two possible sets of operators that commute: {J^2, Jz, J^21z, J^22z} and {J^21z, J^22z, J1z, J2z} What I don't understand is why the...
  4. Hoophy

    Circular Motion With Constant Angular Acceleration

    Homework Statement Question: A 600 g steel block rotates on a steel table while attached to a 1.20 m-long hollow tube. Compressed air fed through the tube and ejected from a nozzle on the back of the block exerts a thrust force of 5.01 N perpendicular to the tube. The maximum tension the tube...
  5. D

    Determine the angular velocity as a function of the angle

    Homework Statement A solid body begins to rotate around a fixed axis with angular acceleration ##\beta=\beta_0\cosφ##, where ##\beta_0## is a constant vector, ##φ##, is the angle of rotation of the body from initial position. Determine the angular velocity of this body as a function of the...
  6. Abhishek11235

    Why is angular momentum conserved?

    So,this is problem from David Morin's Classical Mechanics(Screenshot 1). I solved the problem. Then I went to see the solution in manual hoping for out of box thinking. As in screenshot 2 is solution by Morin. My question is why he conserves angular momentum about the point (R-h) below C.M...
  7. brotherbobby

    Alpha Particle Scattering and angular momentum

    Statement of the problem : "Using the definition L = r ##\times## p, prove that the direction of L is constant for an alpha (##\alpha##) particle whose scattering is shown in the diagram below. " Relevant equations : We are aware that the scattering takes place via a central force F = F(r)...
  8. D

    1D Angular Motion with different velocity stages

    Homework Statement I am solving a kinematic problem, where I have a link that is attached to a rotational joint. I need to find the position of the joint for t=0..8, and I need to do it for every 0.01s. The problem comes from the fact that I have three stages for the velocity, during t = 0..0.1...
  9. R

    I Exploring the Lamb Shift: G-Factor & Angular Momentum

    Good day dear forum, greetings from Argentina. I am studying the Lamb Shift, which says that in the atomic orbitals, an upward energy shift occurs due to an interaction of the electron with itself. This means that a level s can have an energy slightly greater than a level p. So far so good, but...
  10. B

    Angular and Linear Momentum Problem

    Homework Statement A system has a ball and a uniform rod. The rod is rotating about point X on a frictionless table until it strikes the ball. The rod stops and the ball moves away. Variables: Rod's mass: m1 Ball's mass: m2 Rod's original angular velocity: ω Ball's final velocity: v Rod's...
  11. N

    Calculus angular acceleration with respect to theta

    Homework Statement A disk with a 0.4 m radius starts from rest and is given an angular acceleration α = (10θ2/3)rad/s2 , where θ is in radians. Determine the magnitude of the normal (centripetal and tangential components of a point P on the rim of the disk when t = 4s. Homework Equations α =...
  12. U

    MHB What is the relationship between frequency and angular speed?

    Hello there, I'm new here and i need some help on my home work. A DVD drive rotates at an angular frequency of 4800 rpm. a) what is it's angular speed in rpm? b) at 4800 rpm, what is the linear speed (in knm/hr) of (i) the center point and points (ii) 5 cm and (iii) 6 cm from the center? Thanks!
  13. brotherbobby

    Angular momentum of a pendulum

    Statement of the problem : A ball shown in the figure is allowed to swing in a vertical plane like a simple pendulum. Answer the following : (a) Is the angular momentum of the ball conserved? No, the angular momentum ##L = mvl##, where m is the mass of the ball and v is its speed at an...
  14. QuarkDecay

    I C-G coefficients and Angular momentum

    Can someone explain to me how we find it? Examples Y10X-= = |1,0>|1/2,-1/2> = √2/3 |3/2,-1/2> + √1/3|1/2,-1/2> Y11X-= = |1,1>|1/2,-1/2> = √1/3|3/2,1/2> + √2/3|1/2,1/2> Y2-1X- = = √4/5 | 5/2, -3/2> + √1/5 | 3/2, -3/2> I understand it goes like YlmX± = |l,m>|s,ms> = a |jmax, ?> + b |jmin, ?>...
  15. A

    About linear and angular momentum

    Homework Statement Homework Equations For this problem I got the angular momentum conservation equations, mv(l+h)=mv'(l+h)+Ml2ω and momentum conservation equation as mv(l+h)=mv'(l+h) m=colliding mass,v and v' velocity before and after collision. M=mass of the rod. 2l=length of the rod...
  16. A

    How Do You Calculate the Ratio h/R for a Spinning Billiard Ball?

    1. Homework Statement A spherical billiard ball of uniform density has mass m and radius R and moment of inertia about the center of mass ( ) 2 cm I = 2/ 5 mR^2 . The ball, initially at rest on a table, is given a sharp horizontal impulse by a cue stick that is held an unknown distance h above...
  17. A

    Angular frequency of the small oscillations of a pendulum

    Homework Statement One silly thing may be I am missing for small oscillations of a pendulum the potential energy is -mglcosθ ,for θ=0 is the point of stable equilibrium (e.g minimum potential energy) .Homework Equations Small oscillations angular frequency ω=√(d2Veffect./mdθ2) about stable...
  18. U

    How Do You Calculate the Natural Angular Frequency of a Dual-Spring System?

    Homework Statement The suspension of a modified baby bouncer is modeled by a model spring AP with stiffness k1 and a model damper BP with damping coefficient r. The seat is tethered to the ground, and this tether is modeled by a second model spring PC with stiffness k2. The bouncer is...
  19. J

    The conservation of angular momentum

    This question is about the conservation of angular momentum: So far, I have understood the reason as to why an object with a high moment of inertia has a small angular acceleration whereas an object with a low moment of inertia has a larger angular acceleration. The reason for this is that if...
  20. G

    Conservation of angular momentum in scattering processes

    Greetings. So... let us consider a particle moving in the yz plane, coming from the infinite towards a region were a gravitational potential is appreciable. The Lagrangian of the system is \mathcal{L} = \frac{1}{2}\mu (\dot{r}^2+r^2{\dot \phi}^2) + \frac{G\,m\,M}{r} where \mu is the reduced...
  21. E

    Conservation of momentum and angular momentum

    Homework Statement 4 masses attached by a cross with no mass are spinning on a smooth table around the center of the cross. The distance between any mass to the center is L. The angular velocity is ω0. m1=m3,m2=m4 Suddenly, at t=0 (the time described in the picture), m4 disconnects from the...
  22. E

    Angular momentum conservation and center of mass

    Homework Statement Two bodies with an equal mass of M are attached by a pole with no mass with a length of L. The system is placed on a horizontal table and at first it is at rest. At t=0 a bullet with a mass of m hits the pole, as described in the picture. The collision is completely elastic...
  23. olgerm

    Angular speed that breaks a spinning body apart with inertial stress

    How to find the angular speed, on which a spinning hollow cylindrical body breaks due to inertial stress(force)? I found 2 sources(http://www.roymech.co.uk/Useful_Tables/Cams_Springs/Flywheels.html (last 2 equations) ...
  24. G

    I The 1.22 factor in the angular resolution

    Hi. The angular resolution is calculated through $$\theta=1.22\frac{\lambda}{D}\enspace.$$ It's the first zero of the intensity function (in small-angle approximation) of the Airy disk...
  25. F

    Lagrangian for relativistic angular momentum

    Hi everyone, I have a question that can't solve. Does exist a lagrangian for the relativistic angular momentum (AM)? I can't even understand the question because it has no sense for me... I mean, the lagrangian is a scalar function of the system(particle,field,...), it isn't a function FOR the...
  26. WhiteWolf98

    Rigid Bodies/ Angular Velocity

    Homework Statement Homework Equations ##v=\omega r## The Attempt at a Solution So, using the equation, one can work out the velocity at point ##B##. ##v_B=\omega_{AB} \cdot r_B## ##v_B=6(0.4)=2.4~ ms^{-1}##I then tried working out the angular velocity at point ##C## using the instantaneous...
  27. Raphael30

    Calculating Angular Velocity and Tensions After a Ball Hits a Board at an Angle

    For a question involving a ball hitting orthogonally the bottom corner of a board held by wires, I need to calculate the angular velocity of the board and ball (collision is inelastic) right after the collision, before there's any external torque. I can easily calculate the angular momentum L...
  28. F

    B Does galaxy formation conserve mass and angular momentum?

    Last week I posted in General Physics some questions about what happens in a collapsing gas cloud, and I was advised that total angular momentum is conserved. I thought of asking for extra clarification here, as that seems really amazing -- I apologize for asking the same thing twice. I use a...
  29. R

    I Question about one electron hydrogen atom angular moment

    Hi, I'm having trouble understanding angular moment of the one electron hydrogen atom. Solving Schrodinger equation on a referece system (say S) I get the energy eigenstates. They depend on three quantum numbers, n, l, m \frac{-ħ}{2 m}\nabla^{2} \Psi - \frac{e^{2}}{4 \pi \epsilon r} \Psi =...
  30. fight_club_alum

    What is the Angular Speed of a Rod Pivoted at One End?

    Homework Statement A uniform rod (mass = 1.5 kg) is 2.0 m long. The rod is pivoted about a horizontal, frictionless pin through one end. The rod is released from rest in a horizontal position. What is the angular speed of the rod when the rod makes an angle of 30 degrees with the horizontal...
  31. F

    Conservation of Angular Momentum is Dumbfounding

    I find conservation of energy and linear momentum to be quite natural to understand, but I find conservation of angular momentum really, really tricky. Let me give two examples: (a) I call my system as a stick with identical springs at its ends, facing opposite directions, each spring is coiled...
  32. A

    Pendulum & Bullet, Understanding and Applying Angular Momentum

    This took a lot of time and effort and I understand if you wish to skip past everything and just read my questions about it in the The too long didn't read summary (TL;DR) at the bottom. Homework Statement The 10-g bullet having a velocity of v = 750 m/s is fired into the edge of the 6-kg...
  33. M

    Determining Eigenvalues and Eigenvectors in a Coupled 2-Particle System

    Homework Statement Consider a 2-particle system where the two particles have angular momentum operators ##\vec{L}_1## and ##\vec{L}_2## respectively. The Hamiltonian is given by $$H = \mu\vec{B}\cdot (\vec{L}_1+\vec{L}_2)+\gamma \vec{L}_1\cdot \vec{L}_2.$$ Determine explicitly the eigenvalues...
  34. Edge5

    I Angular momentum and spin unit

    I know that spin is a type of intrinsic angular momentum. For electron spin is (1/2)ħ . But unit of (1/2)ħ is J.s, which is not the unit of angular momentum. Can you please explain this discrepancy?
  35. J

    Angular Velocity Problem — a Piston and a Crank

    Homework Statement I have been set this question and I am struggling with parts b and c. I think I am nearly there but can't quite get over the line. Please could someone give me a nudge in the right direction. [/B] 1. (a) For the mechanism shown in FIGURE 1 determine for the angle θ = 45°: (i)...
  36. N

    How to find the minimum angular resolution?

    Homework Statement λ = 240 nm D = 2.40 m We are supposed to find the angle resolution (minimum angular distance between two objects so we still see them as two separate objects) to the telescope for ultraviolet light with wavelength 240 nm Homework Equations The equation that I used is θ =...
  37. christang_1023

    Does the spin angular momentum count?

    Taking the Earth orbiting the sun as an example, when I consider the angular momentum of the Earth about the sun, should the spin angular momentum be counted? I'm confused that if it's counted, the spin angular momentum, Lcm=Icm×ωspin, is different from other angular momentum regarding the...
  38. T

    I Calculating the eigenvalue of orbital angular momentum

    Hello, I'm trying to calculate the measurement of the orbital angular momentum of the state l=1 and m = -1. The operator for the angular momentum squared is ## L^2 = -\hbar (\frac{1}{sin\theta}(\frac{\partial}{\partial \theta}(sin\theta\frac{\partial}{\partial \theta}))...
  39. L

    Torque and Angular Momentum - Origin Misconception

    Homework Statement (Problems/diagrams referenced are attached as images.) Homework Equations Net torque about an origin = time derivative of the angular momentum vector about the same origin. The Attempt at a Solution I've solved these problems before, but I'm now looking back at them and...
  40. M

    Angular Momentum of a Moving Particle

    Homework Statement A point particle travels in a straight line at constant speed, and the closest distance it comes to the origin of coordinates is a distance l. With respect to this origin, does the particle have nonzero angular momentum? As the particle moves along its straight-line path...
  41. C

    I A little doubt regarding specific angular momentum

    Good afternoon I just have this little doubt: imagine the osculating orbit of Mars changing slowly in its elements along the centuries. The semi major axis changes, the period, etc. Is the specific angular momentum allways equal in all the osculating orbits Mars has in those centuries? Or does...
  42. S

    Using the angular momentum principle for 2 pucks

    Homework Statement Two pucks are lying on ice where they can slide and rotate with almost no friction. A string is tied to both pucks but it is tied to the middle of the first puck and wrapped around the second puck. You pull on both strings with the same force, F. The first puck moves without...
  43. D

    MHB Total Angular Momentum of a Tractrix

    I have a 5.0 m tractrix and am trying to work out angular momentum and total angular momentum for two hitchpoint speeds 60 & 70 km/h. My result shows a higher total angular momentum for the lower speed. This is not what I expected. Here are my equations Positions: Derivatives Angular velocity...
  44. C

    Please help finish my deduction for angular momentum

    I'm trying to deduce the angular momentum ( for a rigid body ) on my own, and here is the problem I face. By introducing the angular momentum of a tiny piece in rigid body (" i ") as : Li = ri × pi Li = ri × mi vi --------------------------------- [ Line 1 ] Li = ri × mi ri ωi To find the...
  45. C

    Definition of work done by torque

    I' m trying to derive the work done by a torque from W = ∫ F ⋅ ds and I' ve looked up the internet, it said: W = ∫ F ⋅ ds ( since ds = dθ × r ) ---------------------------------------- ( Line 1 ) it can be written as W = ∫ F ⋅ dθ x r this is a vector triple product , thus can also...
  46. PeroK

    Angular Momentum in Spherical Coordinates

    I've started on "Noether's Theorem" by Neuenschwander. This is page 35 of the 2011 edition. We have the Lagrangian for a central force: ##L = \frac12 m(\dot{r}^2 + r^2 \dot{\theta}^2 + r \dot{\phi}^2 \sin^2 \theta) - U(r)## Which gives the canonical momenta: ##p_{\theta} = mr^2...
  47. Y

    Angular frequency of an ammonia molecule

    Hello 1. Homework Statement The dipole moment of an ammonia molecule is ##d_0=5*10^{-30} C.m##.If we apply a static electric field of ##\mathcal { E }=1*10^{6 }V*m^{-1}## to an ammonia molecule initially in the state ## |ψG⟩## where the nitrogen molecule is considered to be on the left,we make...
  48. Decimal

    I Value of orbital angular momentum for two particles

    Hello, I encountered the following statement in my lecture notes and there is a couple of things I don't understand:"Let's consider two particles with spins ##s_1 = \frac{1}{2}## and ## s_2 = 1## with a spherically symmetric interaction potential. Assume these two particles are in a two...
  49. L

    Simple Ice Skater with Conservation of Angular Momentum

    Homework Statement Not a HW problem, but a "me re-thinking things" problem. Please tell me where my thinking is flawed: You have an ice skater with no net external torques acting on him/her. (We are analyzing the time after they have to get an external torque on them by pushing off of the...
  50. S

    Conservation of angular momentum

    Homework Statement A uniform thin rod AB is equipped at both ends with the hooks as shown in the figure and is supported by a frictionless horizontal table. Initially the rod is hooked at A to a fixed pin C about which it rotates with a constant angular velocity w1 . Suddenly end B of the rod...
Back
Top